Second Derivative of Heaviside Function

Click For Summary
The second derivative of the Heaviside function, H''(x), is considered to be zero everywhere except at x=0, where it is undefined. The discussion highlights the confusion surrounding the definition of the second derivative and its implications for extending a proof involving piecewise-constant functions. It is noted that H'(x) corresponds to the Dirac delta function, δ(x), leading to the question of the nature of δ'(x). Participants clarify that both H' and H'' are not defined at x=0, which affects the validity of certain mathematical approaches. Overall, the consensus is that while H''(x) is zero elsewhere, its behavior at x=0 complicates its application in proofs.
Choragos
Messages
20
Reaction score
0
Hello all. In short, I am wondering what the second derivative of the Heaviside function (let's say H[(0)]) would be. I'm presuming that it's undefined (or more accurately, zero everywhere but at x=0), but I would like to know if that is correct.

Essentially, I am attempting to extend a proof which uses f(x) where f''(x) is defined. I would like to extend this proof to a piecewise-constant f(x). However, one of the requirements is that f'(x) >> f''(x) >> f(n)(x). If H''[(0)] is undefined, than this approach will not work.

My attempt at a solution is as follows. Given a general, differentiable function f(x), then

\intf(x)δ(n)dx \equiv -\int\frac{∂f}{∂x}δ(n-1)(x)dx

I am, however, unclear as to the meaning of δ(n-1), especially when n=1.

Thanks for your help.
 
Physics news on Phys.org
Choragos said:
Hello all. In short, I am wondering what the second derivative of the Heaviside function (let's say H[(0)]) would be. I'm presuming that it's undefined (or more accurately, zero everywhere but at x=0), but I would like to know if that is correct.
The Heaviside function is defined everywhere, but it's not continuous at x = 0. What are you referring to when you say "it's undefined", the Heaviside function or its 2nd derivative? What you wrote isn't clear.

If x < 0, H'(x) = 0. If x > 0, H'(x) = 0, so on these intervals, H''(x) would also be zero.
Choragos said:
Essentially, I am attempting to extend a proof which uses f(x) where f''(x) is defined. I would like to extend this proof to a piecewise-constant f(x). However, one of the requirements is that f'(x) >> f''(x) >> f(n)(x). If H''[(0)] is undefined, than this approach will not work.
Both H' and H'' are not defined at x = 0.
Choragos said:
My attempt at a solution is as follows. Given a general, differentiable function f(x), then

\intf(x)δ(n)dx \equiv -\int\frac{∂f}{∂x}δ(n-1)(x)dx

I am, however, unclear as to the meaning of δ(n-1), especially when n=1.

Thanks for your help.
 
Thanks for your reply. I meant that the second derivative did not exist at x=0.

My notation is sloppy (my apologies, I'm a geophysicist and a poor mathematician), but shouldn't H'=δ, where δ is the Dirac delta function? Proceeding from that, then the question becomes δ'=? That is why I was investigating the identity above, but wasn't really understanding what it was telling me.

I agree that H'' = 0 everywhere but at x=0. I suppose H''(0) is undefined.
 
Choragos said:
Thanks for your reply. I meant that the second derivative did not exist at x=0.

My notation is sloppy (my apologies, I'm a geophysicist and a poor mathematician), but shouldn't H'=δ, where δ is the Dirac delta function?
One definition is
$$ H(x) = \int_{-\infty}^x δ(t)dt$$
"although this expansion may not hold (or even make much sense) for x = 0." http://en.wikipedia.org/wiki/Heaviside_step_function
If you differentiate both sides above, H'(x) = δ(x), using the FTC.

I was look at H(x) from the perspective of being a step function instead of as an integra.
Choragos said:
Proceeding from that, then the question becomes δ'=? That is why I was investigating the identity above, but wasn't really understanding what it was telling me.

I agree that H'' = 0 everywhere but at x=0. I suppose H''(0) is undefined.
 
Question: A clock's minute hand has length 4 and its hour hand has length 3. What is the distance between the tips at the moment when it is increasing most rapidly?(Putnam Exam Question) Answer: Making assumption that both the hands moves at constant angular velocities, the answer is ## \sqrt{7} .## But don't you think this assumption is somewhat doubtful and wrong?

Similar threads

  • · Replies 2 ·
Replies
2
Views
3K
  • · Replies 10 ·
Replies
10
Views
3K
  • · Replies 5 ·
Replies
5
Views
2K
Replies
8
Views
2K
  • · Replies 11 ·
Replies
11
Views
2K
  • · Replies 14 ·
Replies
14
Views
2K
  • · Replies 8 ·
Replies
8
Views
1K
  • · Replies 13 ·
Replies
13
Views
2K
  • · Replies 1 ·
Replies
1
Views
2K
  • · Replies 3 ·
Replies
3
Views
5K